Mathcenter Forum

Mathcenter Forum (https://www.mathcenter.net/forum/index.php)
-   คณิตศาสตร์อุดมศึกษา (https://www.mathcenter.net/forum/forumdisplay.php?f=2)
-   -   ปัญหาชิงรางวัลข้อที่ 5: From Number Theory Marathon (https://www.mathcenter.net/forum/showthread.php?t=1241)

warut 12 มกราคม 2006 07:33

ปัญหาชิงรางวัลข้อที่ 5: From Number Theory Marathon
 
ข้อนี้เอามาจากโจทย์ของคุณ gools ครับ

จงแสดงว่า ถ้า \(a\) และ \(b\) เป็นจำนวนเต็มแล้ว\[\frac{a^2-2}{2b^2+3}\]ไม่เป็นจำนวนเต็ม

ป.ล. ข้อนี้คุณ gools ก็มีสิทธิ์ตอบด้วยนะครับ

Punk 13 มกราคม 2006 21:15

ขอร่วมด้วยคนครับ

ข้อนี้ผมคิดได้แค่บางส่วนคือได้ว่า ถ้า มีจำนวนนับ $a,b$ อย่างที่ว่า แล้ว $b$ ต้องหารด้วย 5 ลงตัว แต่ยังหาข้อขัดแย้งกรณีนั้นไม่ได้

เริ่มจาก (รื้อฟื้นความรู้ Number Theory หลังจากทิ้งไปนานร่วมสิบปี :D ) สมมติว่า มีจำนวนนับ $a,b$ อย่างว่า

1) $a^2-2\equiv-2,-1,2\;\text{mod}5$ ดังนั้น 5 หาร $a^2-2$ ไม่ลงตัว

2) เขียน $2b^2+3=2(b^2-1)+5$ ดังนั้น 5 ต้องหาร $b^2-1$ ไม่ลงตัว ไม่งั้นจะขัดแย้งกับ 1) เพราะฉะนั้น $b-1,b+1\not\equiv0\;\text{mod}5$

3) ทำนองเดียวกันกับข้อ 2) ถ้าเขียน $2b^2+3=2(b^2-4)+10$ จะได้ว่า 5 ต้องหาร $b^2-4$ ไม่ลงตัว เพราะฉะนั้น $b-2,b+2\not\equiv0\;\text{mod}5$

ดังนั้น 5 ต้องหาร $b$ ลงตัว (ได้แค่นี้แหละ จะได้กี่คะแนนเนี่ย :p )

R-Tummykung de Lamar 13 มกราคม 2006 21:46

พี่ Punk ครับ ตรงนี้อะคับ
อ้างอิง:

$$2b^2+3=2(b^2-4)+10$$
ต้องเป็น $2b^2+3=2(b^2-4)+11$ อะครับ :D
(ผมก็ยังคิดไม่ได้เลย ^^)

Punk 13 มกราคม 2006 21:53

จริงด้วย :mellow: ขอบคุณน้อง tummy มากครับ โดนหักคะแนนอื้อแน่เลย

warut 13 มกราคม 2006 22:28

ก่อนอื่นต้องขอโทษด้วยนะครับที่ช่วงนี้ผมไม่ได้แวะเข้ามาเลย เลยยังไม่ได้อ่านอะไรทั้งนั้น แต่ที่สำคัญคือผมอยากจะมาพูดอะไรเกี่ยวกับโจทย์ข้อนี้สักหน่อยนึง

คือที่ผมยกเอาโจทย์ข้อนี้ขึ้นมาก็เพื่อเป็นตัวอย่างของโจทย์จำนวนมากที่เอามาโพสต์กันที่นี่ที่มีลักษณะเด่นคือเป็นโจทย์ที่ยากมากๆ บังเอิญข้อนี้เป็นข้อที่ผมรู้ก็เลยสามารถบอกได้ว่ามันต้องใช้ความรู้เรื่องอะไร ระดับไหน แต่ผมเชื่อว่าโจทย์อื่นๆที่ผมทำไม่ได้แต่เป็นเรื่องที่ผมไม่คุ้นก็คงมีลักษณะแบบนี้เหมือนกัน

ส่วนโจทย์ข้อ 1-4 ของผมนั่น ในส่วนของการคิดคำนวณใช้ความรู้ ม.ปลายทุกอันครับ จะมีก็แต่ข้อ 1 ที่อาจต้องใช้ความรู้รอบตัว(ที่ไม่ได้เกี่ยวกับการคิดคำนวณ)ที่ไม่ได้อยู่ในหลักสูตรด้วย

ดังนั้นมองในแง่ของความน่าจะเป็นน่าจะใช้ความพยายามกับโจทย์ข้อ 1 และ 3 ก่อน แต่อย่างว่าครับ...ลางเนื้อชอบลางยา

gools 16 มกราคม 2006 16:32

ของผมใช้ความรู้เรื่อง Quadratic Residue น่ะครับ
จะพิสูจน์ว่าสมภาค $a^2 \equiv 2 (\bmod (2b^2+3)) $ ไม่มีคำตอบ
ให้ $(a/m)$ เป็นสัญลักษณ์ Jacobi และใช้ความรู้ที่ว่า
\[(2/m)=(-1)^{(m^2-1)/8}\]
เนื่องจาก $(2b^2+3)^2-1=4b^4+12b^2+8=4(b^2+2)(b^2+1)$
ให้ $b \equiv 0,1,2,3 (\bmod 4)$ เราจะได้ว่า $(b^2+2)(b^2+1) \equiv 2(\bmod 4)$
ดังนั้น $16 \not| (2b^2+3)^2-1$
เราจะได้ว่า $(2/2b^2+3)=-1$
ดังนั้นสมภาค $a^2 \equiv 2 (\bmod (2b^2+3)) $ ไม่มีคำตอบ

Edit:แก้ตามที่พี่ nongtum บอกแล้วนะครับ :)

nongtum 16 มกราคม 2006 16:56

คาดไม่ถึงจริงๆครับว่าจะใช้ quadratic rest แก้ข้อนี้ได้ แต่ตรงสูตรที่ยกมามี n โผล่มาได้ไงเอ่ย และก็จาก $(b^2+2)(b^2+1) \equiv 2(\bmod 4)$ สามารถสรุปได้ว่า $(b^2+2)(b^2+1)-1 \equiv 5(\bmod 8)$ (กรณีที่ทางซ้ายเท่ากับ 1 (mod8) จะได้ $8|b^4+3b^2+1$ เกิดข้อขัดแย้ง) ซึ่งพอแทนในสูตรที่ยกมาจะได้ผลลัพธ์เหมือนกันครับ(สงสัยตอนทดก่อนพิมพ์เบลอจัด)
ที่เหลือก็ไม่น่ามีอะไรแล้วครับ :great:

warut 17 มกราคม 2006 06:01

ใช่ครับ...ข้อนี้คงหลีกเลี่ยงเรื่อง quadratic residue ไม่ได้ นั่นคือสาเหตุที่ผมไม่สามารถจัดให้ข้อนี้อยู่ระดับ ม.ปลายได้ แต่ทำนองเดียวกันผมก็ไม่ได้คาดหวังว่าจะต้องใช้อาวุธหนักมากอย่าง Jacobi's symbol :eek: ดังนั้นผมจึงยังมีอีก 5 คะแนนสำหรับผู้ที่พิสูจน์ข้อนี้ได้โดยใช้เพียงความรู้พื้นฐานเกี่ยวกับ quadratic residue ต่อไปนี้

ทฤษฎีบท

ถ้า \(p\) เป็นจำนวนเฉพาะคี่ แล้วเราจะได้ว่าสมการ \(x^2\equiv2\pmod p\) จะมีคำตอบก็ต่อเมื่อ \(p\equiv\pm1\pmod8\)

สำหรับการเขียนสมการ congruence เราสามารถเขียนโดยไม่ต้องพิมพ์เครื่องหมายวงเล็บรอบ modulus ให้ลำบากโดยการใช้ \pmod (p มาจากคำว่า parentheses ครับ) เช่น ถ้าเราเขียนว่า x \equiv a \pmod{100} ก็จะได้\[x \equiv a \pmod{100}\]ซึ่งจะเห็นว่ามีวงเล็บขึ้นมาโดยอัตโนมัติเลยครับ

nongtum 17 มกราคม 2006 10:18

ให้ x,y เป็นจำนวนเต็ม และ $x^2-2\equiv0\pmod{(2y^2+3)}$ โดยอาศัยทฤษฏีที่คุณ warut กำหนดให้จะทำได้ดังนี้

ให้ $2y^2+3=\prod{p_i^{k_i}}$ สำหรับ $k_i\in\mathbb{N}$ และจำนวนเฉพาะ $p_i\ge3$
จาก $2y^2\equiv0\ ,2\pmod8$ และ $\prod{p_i^{k_i}}-3\equiv0,\ \pm2,\ 4\pmod8$ เราสามารถสรุปได้ว่า $\prod{p_i^{k_i}}\equiv\pm3\pmod8$ ...(*)
เนื่องจาก $(2y^2+3)|x^2-2$ ดังนั้น $x^2-2\equiv0\pmod{p_i}$ นั่นคือ $p_i\equiv\pm1\pmod8$ ซึ่งทำให้ $\prod{p_i^{k_i}\equiv\pm1\pmod8}$ เกิดข้อขัดแย้งกับ (*)
###จบการพิสูจน์###
ปล. กรณีที่ $2y^2+3$ เป็นจำนวนเฉพาะรวมอยู่ในนี้แล้ว (EDIT: แก้และย่อต่อได้ครับแต่เดี๋ยวจะโดนหาว่าลอกวิธีทำคุณ warut มา แต่โดยใจความเป็นวิธีเดียวกันครับ)

warut 17 มกราคม 2006 18:47

หลังจากอ่านอย่างละเอียดแล้วคิดว่าการพิสูจน์ของน้อง gools และคุณ nongtum ถูกต้องครับ แม้ยังไม่ค่อยประทับใจผมเท่าไหร่แต่ว่าเห็นว่าข้อนี้ยากก็เอาไปคนละ 5 คะแนนละกันนะ ต่อไปเป็นเฉลยครับ

เนื่องจาก \(2b^2+3\equiv\pm3\pmod8\) แสดงว่าจะต้องมีจำนวนเฉพาะ \(p\equiv3\) หรือ \(-3\pmod8\) เป็นตัวประกอบอยู่ (เพราะถ้าจำนวนจำนวนหนึ่งมีแต่ตัวประกอบเฉพาะที่อยู่ในรูป \(8k\pm1\) จำนวนนั้นก็จะต้องอยู่ในรูป \(8k\pm1\)) แต่โดยทฤษฎีบทที่ผมให้ไว้ข้างบนทำให้เรารู้ว่า \(a^2\not\equiv2\pmod p\) เราจึงได้ว่า \((a^2-2)/(2b^2+3)\) ไม่ใช่จำนวนเต็ม


เวลาที่แสดงทั้งหมด เป็นเวลาที่ประเทศไทย (GMT +7) ขณะนี้เป็นเวลา 19:43

Powered by vBulletin® Copyright ©2000 - 2024, Jelsoft Enterprises Ltd.
Modified by Jetsada Karnpracha